« first day (2014 days earlier)      last day (3014 days later) » 
00:00 - 18:0018:00 - 00:00

12:22 AM
@ForeverMozart I would be interested in cases where people had a significat IQ boost over a certain period of time.
In every test I take (with time) I score higher, and significantly higher.
 
scales are different so you have to be sure you compare correctly
schedule a real IQ test
at your local dept of education
 
@ForeverMozart I know this, but overall, this is what I noticed over the time. Even if the tests I took maybe are not perfectly like the ones that are called to be real, the trend was always to get higher and significantly higher.
 
it may be because you know what patterns to look for on the test, or see similar problems
 
@ForeverMozart yeah, it could be. But I also took test with new questions to me.
 
like many kids take the SAT over and over and their scores improve
 
12:27 AM
@ForeverMozart Yeah, maybe it has no meaning.
 
I'm not sure
I bet if you were sleep deprived your score would be lower
so it depends on lots of things
 
Is $((0,1],d)$, where $d$ is the standard metric, a valid metric space? I.e., does the metric space depend on the set?
 
@ForeverMozart I'm sleep deprived a lot these days. I sleep less and work a huge lot.
It's 2:30 here, late, and I go to sleep evey night around 3:00-4:00 or later.
 
for a metric to make sense, you need to know you can evaluate it at pairs of points from the set
your standard metric $d$ is defined by $d(x,y)=|x-y|$
 
So the only restriction would be that the set be not empty.
 
12:30 AM
for $x,y\in\mathbb R$
 
12:48 AM
Hello everybody.
Just a small question.
Regarding the representations of D10.
Let x be the rotation (of degree 5), and y be the rotation.
Then, say we have an irreducible complex representation of D10.
$\rho: G \to GL(V)$
Now, consider $\rho(x)$.
This is non-zero (since it's invertible, for instance) and as \mathbb{C} is algebraically closed, must have an eigenvector, say, $v$.
Then, $x(v) = \mu v$, say. (I'm suppressing the \rho's here for clarity.)
We must have $\mu^5 = 1$.
Now, consider $y(v)$. This is an eigenvector of $x$ as well, via, $xy(v) = y x^(-1)(v) = \mu^{-1} (y(v))$, with eigenvalue $\mu^{-1}$.
Now, if $y(v) \in span(v)$, then the representation is one dimensional.
And therefore irreducible, and we must have $V = \mathbb{C}$.
But, if $y(v) \not \in span(v)$, then, $span(v, y(v))$ is a G-subspace of $V$ and so, we must have $V = W$.
Therefore, the other irreducible representations must be 2 dimensional.
Now, here's my question: how do we find out which ones these are?
Like, I can find one of them; namely, let x = (\mu 0; 0 \mu^{-1}), y = (0 1;1 0).
But as I understand it, there exists another one of these out there that's non-isomorphic to this one.
Could it be perhaps x = (1 0;0 1), y = (0 1;1 0)?
But that can't be right, because there exist 1D G-invariant subspaces.
Blegh!
 
 
2 hours later…
r9m
2:49 AM
@I'manartist no ploylogs :( .. you've gotta be kiddin me .. well whatever can be managed by integration by parts of basic series transforms can be achieved by summation by parts too .. so I guess I shouldn't be surprised :) Seeing a first-hand calculation that way would be awesome though :D
@I'manartist 'kay! ..
 
 
1 hour later…
4:17 AM
A manufacturer has six distinct motors in stock, two of which came from a particular supplier. The motors must be divided among two production lines, with three motors going to each line. If the assignment of motors to lines is random, find the probability that both motors from the particular supplier are assigned to the first line.
The answer is $1/5$ - what I don't understand is where the $5$ comes from?
 
4:29 AM
Did you draw a tree diagram?
First find the probability that one motor is assigned.
 
4:44 AM
Nvm sleep deprivation is messing with my brain. I would ask it on mse
 
@Michael I've been told the probability is the number of outcomes in which the two from the particular supplier end up on the first line divided by the number of outcomes. However, I don't know how to calculate either of those things. The total number of outcomes would be $6$ in my mind, which is most likely wrong.
 
Do it step by step
I really want to help but I have to go sleep
Ask @Clarinetist
 
@Michael No worries. Have a nice sleep.
 
Oh he isn't here
Uhm
 
I've already asked it in the main forum.
 
4:58 AM
They gave you a hint right?
 
Not yet. I asked only a few minutes ago.
 
They answer super quickly don't worry
 
Yeah, this forum is amazing! I dislike asking questions when I've no reasonable attempt to show, though, which is unfortunately the case with this one.
 
All I can say is the the total number of outcomes is definitely not 6
Suppose it is 6 then we have x/6=1/5
 
According to Wikipedia the permutation matrix of $\pi \in S_n$ is given by $P_\pi$ with $(P_\pi)_{ij} = \delta_{\pi(i),j}$, which apparently results in $P_\pi P_\tau = P_{\pi \tau}$. But if I’m not mistaken we actually get $P_\pi e_i = e_{\pi^{-1}(i)}$, which results in $P_\pi P_\tau = P_{\tau \pi}$. Can anyone find whatever mistake I’m making?
 
5:02 AM
@Michael Indeed that wouldn't make sense. xD
 
So what can the outcome be?
It has to be a multiple of 5
You can use logic or you can use combinatorics
 
Yeah it would be a multiple of five.
 
Ever heard of ncr
Where n is the total event, c is choose, and r is the successes
Total events*
 
Binomial coefficients?
 
Does anyone know about a good book that deals with quotient spaces/maps?
A book in topology that is
 
5:09 AM
@user276387 I would love to explain but I have to go. Tell me if you get it ok? :)
 
@Michael Yeah, alright. Take care. :]
@Michael the answer was binomial coefficients indeed! xD
 
r9m
6:01 AM
Hello @Cody man!
 
6:12 AM
m
please
 
Hello all!
 
What's everyone up to?
 
6:29 AM
I have idea for interesting construction
trying to write it in symbols
very difficult
it is construction of a special topological space
and you?
:27415019
@SAWblade
DUDE
ARE YOU THERE MAN
 
Yes. xD
Watching anime though so distracted. xD
 
are you ASIAN
 
No.
 
oh I thought anime was asian
 
It is.
Tell me about your construction.
 
6:47 AM
oh
wait
if you remove a point from $(0,1)^\omega$, is it still connected?
 
I have absolutely no idea.
I don't even know what $(0,1)^{\omega }$ is. xD
 
oh wait
yes it is
gotta be
lalala
 
7:41 AM
This is by far the fastest I have had a paper cited. Put on arXiv Jan 26th, cited by a paper put on arXiv Feb 5th.
 
@Tobias Nice. How did you find out?
 
@JulianRachman My coauthor on the cited paper noticed
Doesn't seem to have been updated on my Google Scholar profile yet
 
8:18 AM
@JulianRachman I have not read the citing paper thoroughly yet, though it seems to just be a "similar things have been done in..." citation, rather than an actual application of the cited paper.
 
8:29 AM
Ah. I see
 
(Not that this should be so surprising given the short time. It would have been really impressive if someone had found an application plus the time to write it up in such a short time)
even though the person citing us would probably have been able to, as he is a bit of legend by now
 
Haha. He or she sure is.
 
(the citer is Lusztig)
 
Ok. I'll check it out.
 
 
1 hour later…
hhh
9:46 AM
is the only solution here that x equals zero in which case it would be true with all k>0? (studying elementary course in Algebra)
I had somewhat similar ex earlier
where if you mark $x^k:=y$, you get $y^2=y$ and you can use the part 4 for the part 5. By which I haven't so far found any nonzero solution :/
Anyone able to help on the Algebra?
Perhaps I need to consider complex ring?!
 
@r9m that one is available in my book. The way I got that, and trust me, is very surprising. :-)
 
@hhh Note that the earlier exercise has the condition that $x^2 = x$ for all elements in the ring
this one just has you find some power of $x$ such that this power is idempotent
 
hhh
10:06 AM
@TobiasKildetoft

$y+1=(y+1)^2=y^2+2y+1=3y+1=y+1$ so the idempotency satisfied with $k=2$?
 
@r9m no kidding, and in terms of all achievements I ever talked about, no one was a joke. Just wait to see. ;)
 
hhh
where let $x=y+1$.
 
@hhh Your first equality assumes something not assumed in the question
 
hhh
@TobiasKildetoft that power exist? And the power loses the parity $\pm$?
 
@hhh I have no idea what you mean
Why are you assuming that $y+1$ is idempotent?
 
hhh
10:10 AM
@TobiasKildetoft I did not notice that I made such an assumption, did I?
 
@hhh You wrote $y+1 = (y+1)^2$
 
hhh
$x^k=x^{2k}$ where $y=x^k$ and the goal is to find $k>0$ satisfying it.
 
@hhh Right, but that does not mean that you can conclude anything about $y+1$
 
hhh
Choose $x$ in ring R. $x^k$ and $x^k+1$ are in R with some $k>0$ so $y$ and $y+1$ are in R with some $k>0$. Right?
@TobiasKildetoft can you clarify?
 
@hhh Sure, they are in the ring, but so what?
 
hhh
10:23 AM
@TobiasKildetoft consider $R=(\mathbb Z_2, 0,1,*,+)$ where all even to zero and all odd to 1 with any $k>0$. Is this an example of the ring?
 
@hhh There is no "the" ring.
You are considering any finite ring, no further assumptions
 
What good algorithms do you know for Gaussian elimination over GF(2)?
1
Q: Gaussian elimination algorithm performance

Ilya_GazmanI am developing the quadratic sieve algorithm and I reached a new bottle neck: The matrix processing. I been reading quit a lot about this topic and I found many solutions Gaussian elimination: This perhaps the most common approach for this problem. It's running time is $O(N^3)$ above GF (2) M...

 
hhh
I meant "a finite ring R" and let $x\in R$.
 
@hhh Sure, that is an example of a finite ring
 
hhh
@TobiasKildetoft thank you, I think I solved the part 5 by accident by considering the problem in modular arithmetics :)
 
10:27 AM
@hhh You can't solve it by considering anything with modular arithmetic
 
hhh
?
 
@hhh Well, how did you use modular arithmetic for it?
 
r9m
@I'manartist Yes .. eagerly waiting I am .. :D
 
hhh
@TobiasKildetoft The instance "a finite ring R" exist, $x\in R$, is $R=(\mathbb Z_2, 0,1,*,+)$. When $k>0$,

$x^k=x^{2k} = 0 mod 2

Eg.

1^1=1^2=1mod2
2^2=2^4=0mod2
...

this is my candidate solution, is there something wrong here?
 
@hhh You can't just pick an example when you want to show something in general
 
10:40 AM
@r9m hehe. One question: do you doubt such a way is possible? My solution is actually pretty simple and short. You'll see it in my book.
 
r9m
@I'manartist Well in that case you have a short proof of $\sum\limits_{n=1}^{\infty} \frac{H_n^{(2)}}{2^nn^2}$ too (the one I asked about on M.Se and you answered) :-)
 
@r9m Yeah, I know. :-)
 
r9m
@I'manartist Incredible!! :D I need that book now before my brain explodes in excitement :P .. will you keep a separate chapter on Euler-sums (in the book I mean)?
 
hhh
@TobiasKildetoft

Consider the polynomial ring $R[x]$. $Z[x]_2$, an instance of $k Z[x]$, is a subring of $R[x]$, an ideal of $R[x]$. The task is to show that an ideal exist with some k>0 of the polynomial ring $R[x]$.

Is this "something in general"?
 
@hhh I have no idea what you mean here.
 
10:45 AM
@r9m No, my book is structured in a different way. I don't have a chapter dedicated to the Euler-sum, I mean I don't have a chapter called like that, but I have all the crazy stuff from the Euler-sum area. :-)
 
r9m
@I'manartist so you are telling me that all the contents that you intended to be in the book have been added and what left to do are mere formalities of the publication-house? :)
@I'manartist Crazier the better :D
 
@r9m I don't say anything here. Just let it be a surprise. :-)
 
r9m
@I'manartist okay :) sure ..
 
@r9m I'm curious if anyone ever did that before with the given requirements, the series I talked about above.
 
hhh
@TobiasKildetoft I am trying to reformulate the task

"Let R be a finite, but not necessarily commutative ring, and let x\in R. Show that there exists k\in \mathbb N_{>0} such that x^k=x^{2k}$."

in terms of ideals so that I could solve it "in general" by ideal criterion.

Is this wrong "in general" approach here?
 
10:49 AM
@hhh What do you mean in terms of ideals? I don't see any reason to involve ideals here
 
r9m
@I'manartist no clue .. most of what I have seen about that particular series are on MSE itself .. and I haven't seen a 'no integrals' solution on mse
 
hhh
@TobiasKildetoft my algebra book has that "normal subgroups in group theory corresponds to ideals in ring theory"
 
@hhh Sure, but why would that be at all relevant here?
 
@r9m I see. What I saw used a lot of polylogarithms, crazy, ugly manipulations that can be all avoided.
 
Also, that is a very vague version of "correspond"
 
hhh
10:52 AM
@TobiasKildetoft Because R is ring so addition and multiplication are defined in it with identity element 1 and neutral element 0.
 
@hhh Yes, I am aware of what a ring is
 
r9m
@I'manartist well I'll try to see if I can manage anything .. :-)
 
@r9m Great! :-)
 
r9m
@I'manartist btw did you make any progress with that $\int_0^1 \log (1-x+x^2)\log (1+x+x^2)\,dx$ integral I asked you about couple of months back? :)
 
@r9m I didn't continue working on that. It seemed to me it has some closed form with special values (unknown) of the polygamma. Maybe I'm wrong.
 
r9m
10:57 AM
@I'manartist okay .. I have been dead stuck :| could make any useful progress :(
 
@r9m I didn't continue I felt it doesn't have a nice closed-form. If it has a nice closed form, that's amazing.
 
r9m
@I'manartist maybe .. the problem originally came from O. Furdui and I mailed him about it .. he said he posed it as an open problem :)
 
@r9m Really? I didn't know that. Then maybe he doesn't have a solution.
 
r9m
@I'manartist that's what he told me ..
 
@r9m I think it's true. Maybe he needed that result for another important problem.
 
hhh
11:00 AM
@TobiasKildetoft For now I cannot think about any other approach to prove it in general. The variety V(x^k-x^{2k}), the ideal <x> with some k>0 -- perhaps, thinking.
Some polynomial ring criterion needed.
 
@hhh Do you see that there there are some $n,m\geq 1$ with $x^m = x^{m+n}$?
 
r9m
@I'manartist probably .. well product of log integral has been baffling us for a long time with no known algorithm that covers all the cases :|
 
@r9m I have some helpful stuff under research that could improve that, but I still have to work a lot on that.
@r9m Yeah, I know.
 
r9m
@I'manartist :D Awesome!!
 
hhh
@TobiasKildetoft sure
 
11:04 AM
@r9m :D
 
@hhh Ok, can you then show that also $x^{k+n} = x^k$ for all $k\geq m$?
 
@r9m I have to finish some research and do then some shopping. I'm back in a couple of hours.
 
r9m
@I'manartist 'kay :) bye ..
 
@r9m How is life?
 
hhh
@TobiasKildetoft yes
 
11:17 AM
@hhh Ok, so if $n\geq m$ what do you see about the square of $x^n$?
 
hey Rd. I'm slightly awake :)
 
@BalarkaSen Hi
 
Hello!
@Cody iirc, you're Galactus, right?
 
Yep. that's me
 
Nice to see you here.
 
11:20 AM
I have chatted much in this area. Nice to know it's here.
 
Interesting, I don't remember seeing you here.
 
I have NOT chatted much is what I meant to write. Sorry, but this is not a good time for me. I am getting ready for work and just got up. I may have to catch up with you all later. I was responding to r9m's 'hello'.
 
Ah, OK. Sure, have a good day.
 
I noticed several of you all are writing books, so to speak. Compiling problems. One on Euler sums is indeed a good idea.
You all have a good day and we will speak later this evening. OK
 
hhh
@TobiasKildetoft $x^{2n}\geq x^{k+n}=x^k \forall k\geq m, n\geq m$
 
11:30 AM
@hhh $\geq$?
 
Huy
11:48 AM
@MikeMiller: do you have any idea how it makes sense to talk about the translation length of a parabolic isometry? if we take the infimum of $d(x, f(x))$, this will always be zero for parabolic isometries (but not realized), so what could be meant by it? fix some point and then compare? does such a comparison even make sense?
or maybe just use the Euclidean metric and then it makes sense?
 
hhh
@TobiasKildetoft $x^{2n}=x^{k+n}=x^k \forall k\geq m, n\geq m$
 
@hhh well, with $k = n$
 
hhh
@TobiasKildetoft $x^{2k}=x^k \forall k\geq m$
 
@hhh why?
 
hhh
@TobiasKildetoft
Consider $n,m\geq 1$ with $x^m=x^{m+n}$.
With $k\geq m$, $x^{k+n}=x^k \forall k\geq m$.
With $n\geq m$, $x^{2n}=x^{k+n}=x^k \forall k\geq m, n \geq m.$
With $k=n$,

$$x^{2k}=x^k \forall k\geq m.$$
 
12:04 PM
@hhh You can't say both $k=n$ and have a for all $k$ afterwards.
 
r9m
@BalarkaSen well I am alive :) How are you doing?
 
@r9m Good to hear. Well, good. Tomorrow's the last of madhyamik.
Hi @Huy.
 
r9m
@BalarkaSen oh! Awesome! :D Best of luck!!
 
hhh
@TobiasKildetoft With $k=n$,

$$x^{2k}=x^k \exist k\geq m.$$
 
@r9m Thanks.
 
12:07 PM
@hhh No. We are assuming that $n\geq m$.
 
I see that you started with rings, but is this not just slight variation of the proof that every finite semigroup has idempotent. math.stackexchange.com/questions/353028/…
Or maybe I should have linked to the proof at ProofWiki, which seems similar to what you were doing above: proofwiki.org/wiki/…
 
hhh
@TobiasKildetoft With $k=n$,

$$x^{2k}=x^k \forall n\geq m.$$
 
@hhh No, $n$ is fixed
You really need to brush up on how quantifiers work for this
 
hhh
@TobiasKildetoft With $k=n$,

$$x^{2k}=x^k$$
 
@hhh Yes, but why call it $k$ then?
 
hhh
12:14 PM
@TobiasKildetoft With $k=n$,

$$x^{2n}=x^n$$
 
@hhh Why are you introducing a $k$ at all?
 
hhh
@TobiasKildetoft ?
 
@hhh There is no reason to mention a $k$ at all here.
 
hhh
@TobiasKildetoft

Consider $n,m\geq 1$ with $x^m=x^{m+n}$.
With $k\geq m$, $\forall k\geq m \quad x^{k+n}=x^k$.
With $n\geq m$, $\forall n \geq m \exists k\geq m \quad x^{2n}=x^{k+n}=x^k.$
With $k=n$, $\forall n \geq m \exists k\geq m \exists k=n \quad x^{2n}=x^n$ so

$$\forall n \geq m \exists k\geq m \exists k=n \quad x^{2n}=x^n$$

which is equivalent to

$$\exists k\in\mathbb N_{>0} \quad x^k=x^{2k}$$

with a finite ring R and $x\in R$.
 
@hhh Well, you now have it when $n\geq m$. So you need to show that this is sufficient
 
12:34 PM
@hhh What is it that you are trying to prove? Existence of idempotent in a finite ring?
 
@MartinSleziak That any element in a finite ring has a power which is idempotent
 
I see. So this follows from the fact that any finite semigroup has idempotent. (See the links I posted in my previous message.)
Since the set $\{x^k; k=1,2,3,\dots\}$ forms a semigroup with multiplication.
 
@MartinSleziak Right (and in fact, one reduces to this case inside the semigroup for that proof anyway)
 
The proof which does not need any prerequisites is the same as hhh is trying to write down: Show that there is a cycle and play with it until you find an idempotent.
 
hhh
$\forall n \geq m \exists k\geq m \exists k=n \quad x^{2n}=x^n$ is equivalent to

$$\forall k \geq m \quad x^{2k}=x^k,$$

I cannot yet see where the sufficiency, @TobiasKildetoft.
 
12:40 PM
I recall that I found the way this proof is written down in the book by Hindman and Strauss quite nice. And also some answers in the post on main contain quite clever arguments.
 
@hhh There is no for all $n$ here. We have a fixed pair $(n,m)$ with a certain property (namely that $x^{m+n} = x^m$), and we need to show that we can also find such a pair with the added property that $n\geq m$.
 
Now, I see where Tobias is going with this. That seems to be rather natural continuation of the direction in which hhh started the proof.
 
@MartinSleziak I was also the one to start the direction actually
 
I see. I was too lazy to go look too far back in the transcript.
 
hhh
$\forall n \geq m \exists k\geq m \exists k=n \quad x^{2n}=x^n$ is equivalent to $x^{2k}=x^k$ in some finite ring R $x\in R$ where the sufficiency has not been proved yet.
 
12:49 PM
@hhh Stop putting quantifiers at all.
 
Is it correct to pronounce $a\ge b$ as "a is greater than b" and $a>b$ as "a is strictly greater than b"?
It's the first one I'm worried about
 
hhh
@MartinSleziak but the proofs are about semigroups, not about rings, tea pause :)
 
1:06 PM
@hhh Well, if $R$ is a ring then $(R,\cdot)$ is a semigroup.
So if you have some result which is true for any semigroup/finite semigroup; then the same result holds for $(R,\cdot)$ if $R$ is any ring/finite ring.
 
1:51 PM
What are the prerequisites for algebraic number theory?
 
I am not sure to which extent it answers your question, but there is this post on the main site: Pre-requisites needed for algebraic number theory.
 
2:13 PM
@BalarkaSen Can you help me with a problem from rudin which I told you that I am doing?
 
@Albas I don't know much analysis, but you can ask away anyway.
"Just ask; don't ask to ask".
 
2:30 PM
It is a very fascinating question to me.
Associate to each sequence $a=\{\alpha_n\}$ in which $\alpha_n$ is 0 or 2, the real number $$x(a)=\sum_{n=1}^{\infty}\frac{\alpha_n}{3^n}$$. Prove that the set of all $x(a)$ is precisely the cantor set@BalarkaSen
I can see that we have $3^n$'s in the denominator so intuitively this should be somehow related to the cantor set. But how should I prove it
 
Right. This is a good problem, because you can prove using that that there is a surjection from the cantor set to $[0, 1]$.
@Albas Think about what the Cantor set is.
 
So I have find that surjection and show that $x(a)$ is somehow related to the surjection(here I mean the surjection you talked about between the cantor set and [0,1]?@BalarkaSen
 
I have no idea what that is supposed to mean.
No, what I said is a different fact.
This doesn't have anything to do with the above problem. You can solve the one above by thinking about the middle third construction explicitly.
 
Okay Thanks , Let me try. And I am trying my best not to cheat from anywhere.
Bingo :P
 
Nice to see you're still working on math.
 
2:39 PM
Can you delete that message of yours?
 
Done.
 
Okay got to go. Thanks again. Have to do this question
 
Have fun.
 
3:11 PM
Hi!
Given the joint distribution P(X < x, Y < y), how can I compute P(X+Y < t) ? (X, Y are continuous random variables with density)
 
hhh
What is sufficient to prove that $A=\{k+l\sqrt 3 \mid m,n\in\mathbb Z\}$ is closed under multiplication?

Some condition with ideal $\langle A\rangle$?

Or (k+l\sqrt 3) (k+l\sqrt 3) = k^2+kl\sqrt 3+(l\sqrt 3) k +3*l^2 =K+L\sqrt 3 with some K,L\in\mathbb Z and then considering all linear combinations?
 
Oh you seem quite angry @robjohn
 
@Basj I do?
 
@robjohn your avatar
 
$$ \int \partial_YP(X\lt t-y,Y\lt y)\,\mathrm{d}y $$
 
3:19 PM
@robjohn Yes but here I'm having trouble to determine this
Let me show you:
Here (thm 6.18) we have the joint law for $m_t = inf_{s \leq t} B_s$ and $M_t = sup_{s \leq t} B_s$
ie we have $P(m_t > a, M_t < b)$
 
@Basj That page is unavailable
 
From this I'd like to compute P(M_t - m_t < t)
 
What does $]0, \infty[$ mean? I don't understand why the brackets are facing the opposite way to the usual convention?
 
@Riggs It doesn't contain $0$ or $\infty$
 
@robjohn here is the page: gget.it/skjvj18x/2.jpg
 
3:23 PM
@Balarka A kind of observation:
$$\sum_{n=1}^{\infty}\frac{\alpha_n}{3^n}=\frac{\alpha_1}{3}+\frac{\alpha_2}{3^2}+\frac{\alpha_3}{3^3}+...$$ . Now if I set $\alpha_2$=2 and the rest as $0$ I get $\frac{1}{3}$. Now similarly if I set $\alpha_1=2$ and the rest all $0$ I get $\frac{2}{3}$. This is just, when for the first time $(\frac{1}{3},\frac{2}{3})$ is removed from $[0,1]$. Similarly we can go for the other end points of the intervals.
 
@Basj $$ \int \partial_1P(m_t\gt t-b,M_t\lt b)\,\mathrm{d}b $$
 
@robjohn so do you think I should differentiate inside the series and that's all ?
 
@robjohn ....but what does it contain?
 
@Basj the series? what series?
 
@Balarka I am getting all the end points. Now I am thinking does the cantor set have points except the end points of the intervals? I would have to do something for them then
 
3:29 PM
@Riggs everything between $0$ and $\infty$
 
@robjohn : the one here : gget.it/skjvj18x/2.jpg
 
@Basj one would take the derivative with respect to $a$ term by term.
Then plug in $a=t-b$ and integrate
 
I'll try !
@robjohn I don't really understand what he means by $B_t \in dx$
 
@Huy: What don parabolic Isometries look like again?
 
3:56 PM
Just wanted to brag about a magic answer to an integral: math.stackexchange.com/a/1646174/259262
 
@Basj I don't know. I assume that is covered by the $\mathrm{d}x$ in the formula.
 
Huy
4:17 PM
@MikeMiller those are the ones with exactly one fixed point on the boundary of $\mathbb{H}^2$ and they are conjugate to $z \mapsto z \pm 1$
 
4:48 PM
@DanielFischer hi, I have two linear maps $A,B$, if I define $B\circ A$ it's a bilinear form (we take three normeds space E,F,G). It's continuous and norm smaller than $1$. In my course they said that the norm is equal to $1$ but it's not trivial. Do you know how can I prove this ?
 
5:08 PM
@Albas Right.
 
@Huy: Iunno. Maybe you're supping over d(x,f(x))?
 
@Albas It does have more points than the endpoints at each level.
But you're on the right track.
 
5:25 PM
@BalarkaSen Well I have a feeling that all numbers in the cantor set have a relation with the number 2 but I have loads of physics left to do so I have to leave this for tomorrow
 
hhh
Does symmetry group of circle mean something more than just complex numbers?
 
@Balarka there is this book recommendation that I want from you. What could be a good book for number theory that also deals a bit with transcendental number theory?(Ireland and Rosen(sure making some spelling mistake somewhere)?)
 
Hi @BalarkaSen
 
5:40 PM
Hi @iwriteonbananas, @Danu.
@Albas Ireland & Rosen is algebraic number theory. Have a look at Niven-Montgomery-Zuckermann.
 
Ever heard of 'bordism invariance theorem' or something like that? It says closed $4n$-dimensional manifolds which occur as the boundary of an oriented closed $(4n+1)$-fold have zero signature.
 
Yeah, I have heard of something of that sort. But I don't know what signature is.
 
Signature of a symmetric nonsingular bilinear form is #positive eigenvalues - # negative eigenvalues
 
@hhh Depends on the meaning of symmetry group of circle.
 
Signature of a $4n$-manifold is signature of the Poincare pairing or whatever it's called
 
5:43 PM
@iwriteonbananas Oh, do you mean the cup product form there?
 
Exactly
 
Right.
 
the one $H^{2n}\times H^{2n}\to \Bbb R$
 
hhh
 
Right-o.
 
5:44 PM
(using $\Bbb R$-coeff's)
 
hhh
@BalarkaSen I think this must be a general question
 
I don't understand what your qn is @hhh. The answer to the one above is rotation by $2\pi/n$.
 
hhh
So I need to find out general definition for the symmetry group of a circle
 
$U(1)$
 
5:46 PM
$\cong SO(2)$
oh, sorry, did I just spoiler something?
 
Wait, what?
$U(1)$ is the circle.
You're looking at the set of self-homeoms of $S^1$.
Or self-isometries. Whatever. But $U(1)$ is not what is meant in the question.
 
Well, AFAIK $O(2)$ (forgot reflections earlier) is the symmetry group of $S^1$, no?
$U(1)$ is "close enough" for physicists :)
 
If isometries are meant, yes.
If homeoms are meant, no.
 
Sure, I'm not going to think about homeos
 
U(1) is just the circle itself, so I have no idea how that has anything to do with symmetries of S^1.
 
5:49 PM
Well
 
But then I don't know anything :)
OK, gotta go.
 
$U(1)\cong SO(2)$ and $SO(2)$ is almost (up to reflections) the right thing
 
I believe you.
 
I got a good grade in algtop btw :)
(1.3, where 1 is best and 5 is fail, and 0.3 is the smallest increment possible)
I was most clueless on the following question: Let $X$ have finite fundamental group. Show that every $f:X\to S^1$ is nullhomotopic.
The induced map on $\pi_1$ is trivial and then I thought I could somehow just homotope the entire image using the homotopies on paths but I heard that one needed some little bit of covering space theory, and I have no idea how to do that.
Any hints?
hi @MikeMiller :)
 
@Danu: Because the map is zero on fundamental groups, you can factor that map through the universal cover of $S^1$, which is contractible.
 
5:53 PM
Ah...
I was quite lost in the last few lectures (on covering spaces), so I guess that'd explain that I have no idea why that's true.
 
@Danu: Of course, your argument can't possibly work, or it would imply that any map zero on $\pi_1$ is null-homotopic.
 
True story
(which doesn't sound absolutely absurd to me, but I guess it should!)
 
Uh... so there are no maps $S^2 \to S^2$?
 
Maybe they're all null-homotopic! :D
 
00:00 - 18:0018:00 - 00:00

« first day (2014 days earlier)      last day (3014 days later) »